LSAT and Law School Admissions Forum

Get expert LSAT preparation and law school admissions advice from PowerScore Test Preparation.

 Administrator
PowerScore Staff
  • PowerScore Staff
  • Posts: 8917
  • Joined: Feb 02, 2011
|
#47134
Complete Question Explanation
(The complete setup for this game can be found here: lsat/viewtopic.php?t=2909)

The correct answer choice is (E)

If none of the windows contains O and R, then from the second rule two of the windows contain R, and O must be used in the remaining window:
D10_game #2_#13_diagram 1.png
Of course, from the fourth rule each window must contain O or P, and so the two windows with R (and thus not O) must both contain P:
D10_game #2_#13_diagram 2.png
One RP window must contain G in order to comply with the first rule, and the other RP window must contain Y because Y must be used and every time Y is used, P is used:
D10_game #2_#13_diagram 3.png
With one of the windows established as R, P, and Y (and containing no other colors since due to the third rule G and O cannot be used in that window), answer choice (E) must be correct.

As an aside, at least G or P must used in the “third” window.
 mahsan
  • Posts: 19
  • Joined: Oct 01, 2018
|
#78001
Hello,

The last step in the explanation above states that one RP window must contain G in order to comply with the first rule. While that could be true, must it be true?

Couldn't we instead have:

1: O G P
2: R Y P
3: R Y P

This would still satisfy the first rule and not violate any others unless I'm missing something here.

Appreciate any help with this. Thanks!
 Paul Marsh
PowerScore Staff
  • PowerScore Staff
  • Posts: 290
  • Joined: Oct 15, 2019
|
#78797
Hey mahsan! Nice spot, you're right. We could have that GP block go with the O instead. But either way, we still end up with a RYP window. So our answer stays the same.

Nice one!

Get the most out of your LSAT Prep Plus subscription.

Analyze and track your performance with our Testing and Analytics Package.